LSAT and Law School Admissions Forum

Get expert LSAT preparation and law school admissions advice from PowerScore Test Preparation.

 Administrator
PowerScore Staff
  • PowerScore Staff
  • Posts: 8917
  • Joined: Feb 02, 2011
|
#23677
Complete Question Explanation

Parallel Reasoning-SN. The correct answer choice is (D)

The argument illogically concludes that since physicians who do not answer questions are not competent, a particular physician who does answer questions is certainly competent. You can diagram the "reasoning" as follows:
  • Premise: ..... Not Answer ..... :arrow: ..... Not Competent

    Conclusion: ..... Answer ..... :arrow: ..... Competent
That is a Mistaken Negation, which you can also tell from the linguistic structure of the sentence without actual diagramming, because the sentence preserves the exact flow in negation, rather than negating and reversing the flow into a contrapositive.

You are asked to parallel the flaw, so you need a similar Mistaken Negation.

Answer choice (A): This choice involves perfectly good reasoning, and is therefore incorrect. Meredith is accustomed to compromise, and the condition given is:
  • Large Family ..... :arrow: ..... Accustomed to Compromise
It is therefore reasonable to conclude that Meredith might be from a large family, because she has fulfilled a necessary condition, which establishes temporary possibility.

Answer choice (B): This choice offers "Not In Favor ..... :arrow: ..... Poorly Informed" and concludes that since Leanne is not in favor, she is ill informed. That is a perfectly reasonable application of a the statement, so this choice is wrong.

Answer choice (C): This choice has no conclusion, and therefore cannot reflect the reasoning in the stimulus.

Answer choice (D): This is the correct answer choice. The argument is that since having more than one job prevents one from finding balance, Maggie, who has only one job, must have balance. Similarly to the stimulus, that negates the material without reversing the flow, and is therefore a Mistaken Negation.
  • Premise: ..... More than One Job ..... :arrow: .....No Balance

    Conclusion: ..... Not more than One Job ..... :arrow: .....Balance
Answer choice (E): This choice involves a compound sufficient condition, so is incorrect. The flaw involved in this choice is that the sufficient condition for failure in business involves hot temper and strong will, but Jeremy is granted only one of those qualities, which is not enough to trigger the sufficient condition.
  • Premise: ..... Hot Temper
    ..... ..... ..... AND ..... ..... :arrow: ..... ..... Fail at Business
    ..... ..... ..... Strong Will

    Conclusion: ..... Strong Will ..... :arrow: ..... Fail at Business
This is not, however, a Mistaken Negation.
 jrc3813
  • Posts: 53
  • Joined: Apr 16, 2017
|
#36358
When reading the stimulus I read it as the contrapositive:

Competent :arrow: Answer questions. So I interpreted the error as a mistaken reversal rather than a mistaken negation, but couldn't find the right answer choice because none of the answers involved a mistaken negation. But because D is the only one that involves the that type of conditional flaw I was able to get the right answer. But do you ever see a parallel flaw question where there is one answer choice that has a mistaken negation, and another with a mistaken reversal. Since both are the same kind of mistake, but one involves the contrapositive wouldn't both be technically correct?
 nicholaspavic
PowerScore Staff
  • PowerScore Staff
  • Posts: 271
  • Joined: Jun 12, 2017
|
#36407
jrc3813 wrote:When reading the stimulus I read it as the contrapositive:

Competent :arrow: Answer questions. So I interpreted the error as a mistaken reversal rather than a mistaken negation, but couldn't find the right answer choice because none of the answers involved a mistaken negation. But because D is the only one that involves the that type of conditional flaw I was able to get the right answer. But do you ever see a parallel flaw question where there is one answer choice that has a mistaken negation, and another with a mistaken reversal. Since both are the same kind of mistake, but one involves the contrapositive wouldn't both be technically correct?
Hi jrc,

Excellent question. :-D And always remember that a Mistaken Negation and a Mistaken Reversal are merely contrapositives of one another.

To answer your question, there can be Parallel Reasoning Questions that have the two "Mistaken" answer choices containing each one, respectively. So your job on the test for these types of questions is to precisely mimic the flaw in the stimulus. It's for that reason that when you diagram, you should always stick closely to the language and in this case the negations, so that you can accurately diagram everything fully in order to sucessfully answer the question.

Here, the Patient's premise is that:

Answer :arrow: Competent

and she concludes that

Answer :arrow: Competent

So always attempt to fully diagram (either on paper or in your head) the argument presented in the stimulus because it will be an incredibly important tool for recognizing the exact parallel structure and argument in the correct answer choice.

Thank you for the great question!
 Boudreaux
  • Posts: 9
  • Joined: Jan 04, 2019
|
#62083
Exactly how does the final sentence get diagrammed? It's "that is why I feel confident about my physcian's competence: she carefully answers everyone of my questions, no matter how trivial." I have a feeling that the answer has something to do with the colon, but I'm not sure what that something is.

I diagrammed it as comp :arrow: answer which would have been the CP of the premise.

Thanks
Nick
 Jay Donnell
PowerScore Staff
  • PowerScore Staff
  • Posts: 144
  • Joined: Jan 09, 2019
|
#62097
Hi Nick!

I think the issue here is the confounding nature of the classic LSAT battle: English vs Logic.

The last sentence contains both the conclusion and relevant premise to the flawed argument. Before attempting to diagram a statement as conditional, we must first break the claims up between premise and conclusion.

The given rule from the first sentence should be diagrammed as:

~APQ --> ~CP (not answer patient's questions ----> not a competent physician)


The conclusion in the first half of the last sentence implies that the author is confident of their physician's competence, and the second half (post colon) gives the reason: they carefully answer every question.

This means the argument structure is as such:


~APQ --> ~CP

APQ

__________

CP



That means the argument is based off of the misconstrued principle of: APQ-->CP, which is the Mistaken Negation of the given rule, and the source of the flawed reasoning.


I hope that helps!
 Boudreaux
  • Posts: 9
  • Joined: Jan 04, 2019
|
#62106
Thanks Jay.

I just returned to the question and see that I didn't correctly identify the conclusion. Take care.
 dandelionsroar
  • Posts: 27
  • Joined: Oct 18, 2018
|
#71781
Hello,

Isn't answer A a mistaken reversal and therefore faulty logic? Considering the stem has a mistaken negation, I'm guessing D is the better answer even though A somewhat parallels the flaw as well since reversal and negation are contrapositives?
 Adam Tyson
PowerScore Staff
  • PowerScore Staff
  • Posts: 5153
  • Joined: Apr 14, 2011
|
#71845
Actually, dandelions, answer A is valid logic! The use of the word "might" is what saves it. The author isn't saying "the necessary condition occurred, so the sufficient condition MUST have occurred" (which would be a Mistaken Reversal.) She's saying "the necessary condition occurred, so the sufficient condition COULD have also occurred." That's true! The presence of the necessary condition DOES at least mean the sufficient condition is possible, even if we cannot prove it to be true.
User avatar
 fortunateking
  • Posts: 31
  • Joined: Jan 10, 2022
|
#93346
A is wrong not because it doesnt use the exact form of logic. It's because the difference on certainty level.
Feel confident about=must, while A use the word might, hense wrong.
User avatar
 fortunateking
  • Posts: 31
  • Joined: Jan 10, 2022
|
#93347
Remeber this is a parallel question.

Get the most out of your LSAT Prep Plus subscription.

Analyze and track your performance with our Testing and Analytics Package.